LSAT and Law School Admissions Forum

Get expert LSAT preparation and law school admissions advice from PowerScore Test Preparation.

 Administrator
PowerScore Staff
  • PowerScore Staff
  • Posts: 8916
  • Joined: Feb 02, 2011
|
#27102
Complete Question Explanation
(The complete setup for this game can be found here: lsat/viewtopic.php?t=11371)

The correct answer choice is (D)


In this Cannot Be True question, each of the four incorrect answer choices could be true. Answer choice (D) cannot be true because U and Y must each repair exactly two machines. Therefore, (D) is the correct answer choice.

Answer choice (A) could be true and is thus incorrect because S could be the only technician to repair one machine.

Answer choice (B) could be true and is thus incorrect because S and W, or S and X, could be the only two technicians to repair exactly one machine.

Answer choice (C) could be true and is thus incorrect because S,W, and X could be the only technicians to repair exactly one machine.

Answer choice (E) could be true and is thus incorrect because U, Y, and W, or U, Y and X could be the only three technicians to repair exactly two machines.

Get the most out of your LSAT Prep Plus subscription.

Analyze and track your performance with our Testing and Analytics Package.